Jump to content

sanghamhoc's Content

There have been 76 items by sanghamhoc (Search limited from 25-05-2020)



Sort by                Order  

#692807 $\sqrt[3]{2+\sqrt{n}}+\sqrt[3]{2...

Posted by sanghamhoc on 10-09-2017 - 19:29 in Số học

Giả sử $\sqrt[3]{2+\sqrt{n}}+\sqrt[3]{2-\sqrt{n}}=x(x\in Z^{+})$

$\Rightarrow 4+3x\sqrt[3]{4-n}=x^{3}$ 

Do x nguyên nên $\sqrt[3]{4-n}$ nguyên

$\Rightarrow \frac{x^{3}-4}{3x}\in Z$

$\Rightarrow x\in\begin{Bmatrix} 1;4 \end{Bmatrix}$

$\Rightarrow n\in \begin{Bmatrix} 5 \end{Bmatrix}$

$\Rightarrow \frac{x^{3}-4}{3x}\in Z$

$\Rightarrow x\in\begin{Bmatrix} 1;4 \end{Bmatrix}$ sao ra được 2 cái 1;4 vậy ạ? Anh làm rõ chỗ này giùm e với 




#692762 Tìm k

Posted by sanghamhoc on 10-09-2017 - 05:17 in Số học

Gọi A là tập tất cả các số tự nhiên lẻ không chia hết cho 5 và nhỏ hơn 30. Tìm số k nguyên dương nhỏ nhất sao cho mỗi tập con của A gồm k phần tử đều tồn tại hai số, mà số này chi hết cho số kia.




#692675 $\sqrt[3]{2+\sqrt{n}}+\sqrt[3]{2...

Posted by sanghamhoc on 09-09-2017 - 12:34 in Số học

Tìm tất cả các số nguyên dương n sao cho : $\sqrt[3]{2+\sqrt{n}}+\sqrt[3]{2-\sqrt{n}}$ là số nguyên dương




#692637 $1+\sqrt{1-y^{2}}=y(1+2\sqrt{1-x^...

Posted by sanghamhoc on 08-09-2017 - 21:17 in Phương trình - hệ phương trình - bất phương trình

Mình có cách này không biết có được không

Ta có : VT>0 -> VP>0 $\rightarrow y> 0$

Từ điều kiện của 2 căn thức trong PT 1 ta có $\left\{\begin{matrix}-1\leq x\leq 1 & & \\ 0< y\leq1 & & \end{matrix}\right.$

$\rightarrow xy\leq 1$

 $PT(2)\Leftrightarrow$

$\frac{1}{1+x}+\frac{1}{1+y}+\frac{2}{\sqrt{(1+x)(1+y)}}=\frac{4}{1+\sqrt{xy}}$

Với mọi $xy\leq 1$ , 

Ta có BĐT phụ sau $\frac{1}{1+x}+\frac{1}{1+y}\leq \frac{2}{1+\sqrt{xy}}$

$\Leftrightarrow \frac{1}{1+x}-\frac{1}{1+\sqrt{xy}}+\frac{1}{1+y}-\frac{1}{1+\sqrt{xy}}\leq 0$

$\Leftrightarrow \frac{(\sqrt{y}-\sqrt{x})^{2}(\sqrt{xy}-1)}{(1+x)(1+y)(1+\sqrt{xy})}\leq 0$ (đúng do $xy\leq 1$ )

Áp dụng bđt Bunhiacopxki , ta có $\sqrt{(1+x)(1+y)}\geq 1+\sqrt{xy}$

$\rightarrow PT(2) VT^{2}\leq VP^{2}\rightarrow VT\leq VP$ dấu =xảy ra $\rightarrow$  x=y=1 

$\frac{1}{1+x}+\frac{1}{1+y}\leq \frac{2}{1+\sqrt{xy}}$ Chị ơi cái này cm sao




#692599 $x_{n+1}=\frac{x_{n}}{2+\sqrt{3+x_{n}^{2}}}$

Posted by sanghamhoc on 08-09-2017 - 12:59 in Số học

Cho dãy số ($x_{n}$) được xác định bởi công thức:

$\left\{\begin{matrix} x_{1}=1\\ x_{n+1}=\frac{x_{n}}{2+\sqrt{3+x_{n}^{2}}} \end{matrix}\right.$ $\forall n\in N^*.$

Tìm công thức số hạng tổng quát của $x_{n}.$




#692598 $1+\sqrt{1-y^{2}}=y(1+2\sqrt{1-x^...

Posted by sanghamhoc on 08-09-2017 - 12:42 in Phương trình - hệ phương trình - bất phương trình

Giải hệ phương trinh:

$\left\{\begin{matrix} 1+\sqrt{1-y^{2}}=y(1+2\sqrt{1-x^{2}})\\ \frac{1}{\sqrt{1+x}}+\frac{1}{\sqrt{1+y}}=\frac{2}{\sqrt{1+\sqrt{xy}}} \end{matrix}\right.$

 




#666250 Max,Min của S_{ABCD}

Posted by sanghamhoc on 30-12-2016 - 18:13 in Hình học phẳng

Cho (O;r). M là điểm trong đường tròn khác O.2 dây AC và BD thay đổi luôn đi qua M và vuông góc với nhau.Tìm Max,Min của $S_{ABCD}$




#666241 $A_{1}M=B_{2}N$

Posted by sanghamhoc on 30-12-2016 - 15:22 in Hình học phẳng

Cho 2 đường tròn tâm $O_{1},O_{2}$ không cắt nhau ( không chứa nhau ). $A_{1}A_{2},B_{1}B_{2}$ là các tiếp tuyến chung ngoài của chúng.Đường thẳng $A_{1}B_{2}$ theo thứ tự cắt $O_{1}O_{2}$ tại M,N.Chứng minh: $A_{1}M=B_{2}N$




#666240 $AA_{1},BB_{1},CC_{1}$ đồng quy

Posted by sanghamhoc on 30-12-2016 - 14:55 in Hình học phẳng

Cho tam giác ABC: 1 đường tròn cắt BC,CA,AB lần lượt tại $A_{1},A_{2};B_{1},B_{2};C_{1},C_{2}$.Chứng minh rằng  $AA_{1},BB_{1},CC_{1}$ đồng quy khi và chỉ khi $AA_{1},BB_{1},CC_{1}$ đồng quy




#666057 Tìm điều kiện hcn để tam giác BMN vuông cân

Posted by sanghamhoc on 28-12-2016 - 17:00 in Hình học phẳng

Cho hcn ABCD, kẻ BK vuông góc với AC.Gọi M,N là trung điểm của AK,CD.Biết BM vuông góc với MN. Tìm điều kiện hcn để tam giác BMN vuông cân 




#666056 $\widehat{AIC}=90$

Posted by sanghamhoc on 28-12-2016 - 16:53 in Hình học phẳng

Cho hình vuông ABCD. E,F thỏa $\overrightarrow{BE}=\frac{1}{3}\overrightarrow{BC}$,$\overrightarrow{CF}=-\frac{1}{2}\overrightarrow{CD}$.I là giao điểm AE và BF.Cm: $\widehat{AIC}=90$




#666055 Chứng minh $\widehat{BIC}=90$

Posted by sanghamhoc on 28-12-2016 - 16:40 in Hình học phẳng

Cho $\triangle ABC$ đều.M,N thỏa $\overrightarrow{BN} =\frac{1}{3}\overrightarrow{BC}$;$\overrightarrow{AN}=\frac{1}{3}\overrightarrow{AB}$.I là giao điểm AM và CN .Chứng minh $\widehat{BIC}=90$




#665965 $a.MA^{2}+b.MB^{2}+c.MC^{2}$ đạt Min

Posted by sanghamhoc on 26-12-2016 - 23:00 in Hình học phẳng

Dạ thế còn ý tính min theo a,b,c thì sao ạ ?




#665733 $a.MA^{2}+b.MB^{2}+c.MC^{2}$ đạt Min

Posted by sanghamhoc on 24-12-2016 - 16:21 in Hình học phẳng

Xác định vị trí của M trong tam giác ABC để $a.MA^{2}+b.MB^{2}+c.MC^{2}$ đạt Min. Tính Min đó theo a,b,c




#664582 $\cot C =2(\cot A + \cot B)$

Posted by sanghamhoc on 13-12-2016 - 21:50 in Hình học phẳng

Mọi người giúp e 1 số bài tập về vector với nhé :
1.Cho đường tròn tâm O với 2 dây cung AB,CD cắt nhau tại M.Qua trung điểm F của BD, kẻ FM cắt AC tại K.
Cmr : $\frac{AM^{2}}{CM^{2}}=\frac{AK}{CK}$
2.Cho tam giác ABC, I là tâm đường tròn nội tiếp, G là trọng tâm, IG vuông góc IC. Cm : $\frac{a+b+c}{3}=\frac{2ab}{a+b}$
3. O là tâm đường tròn nội tiếp tam giác ABC, CM: $\frac{OA^{2}}{bc}+\frac{OB^{2}}{ac}+\frac{OC^{2}}{ab}=1$ (a,b,c là 3 cạnh tam giác)
4.Cho tam giác ABC đều tâm O, M là điểm bất kì trong tam giác.Kẻ MD,ME,MF vuông góc với các cạnh CB,CA,AB.Cmr : $\overrightarrow{MD}+\overrightarrow{ME}+\overrightarrow{MF}=\frac{3}{2}\overrightarrow{MO}$
5.Trong tam giác ABC, cho 2 trung tuyến AA',BB' vuông góc với nhau.Cm: $\cot C =2(\cot A + \cot B)$




#664114 $4\overrightarrow{MA}-2\overrightarrow{MB}...

Posted by sanghamhoc on 07-12-2016 - 22:48 in Hình học phẳng

Cho hv ABCD cạnh = a. điểm M bất kì .Cm $4\overrightarrow{MA}-2\overrightarrow{MB}-2\overrightarrow{MC}-\overrightarrow{MD}$ không phụ thuộc vào M




#663236 Min : $\frac{x^{3}}{(y+z)(y+2z)}+...

Posted by sanghamhoc on 27-11-2016 - 22:13 in Bất đẳng thức và cực trị

Áp dụng bđt AM-GM thôi

Anh chỉ rõ cho em được không...e không rõ cái này lắm




#663233 Min : $\frac{x^{3}}{(y+z)(y+2z)}+...

Posted by sanghamhoc on 27-11-2016 - 21:54 in Bất đẳng thức và cực trị

Cho các số thực dương thỏa mãn xyz=8. Tìm Min của  : $\frac{x^{3}}{(y+z)(y+2z)}+\frac{y^{3}}{(z+x)(z+2x)}+\frac{z^{3}}{(x+y)(x+2y)}$




#661990 $B=\frac{1}{630}x^{9} -\frac...

Posted by sanghamhoc on 15-11-2016 - 12:25 in Số học

Câu 1 Cho p là số nguyên tố lẻ, cm : $\sum_{k=1}^{p} C _{p}^{k} C_{p+k}^{k}-(2^{p}+1)\vdots p^{2}$
Câu 2: Cmr $\forall$ số tự nhiên n, phân số sau đây tối giản : $\frac{21n+4}{14n+3}$
Câu 3:$B=\frac{1}{630}x^{9} -\frac{1}{27}x^{7}+\frac{13}{30}x^{5}-\frac{82}{63}x^{3}+\frac{32}{35}x$
Cmr $B\in Z \forall x\in Z$

Câu 4: Cmr với mọi số nguyên a;b
(3a+5b;8a+13b)=(a;b)




#661989 $\frac{21n+4}{14n+3}$ tối giản

Posted by sanghamhoc on 15-11-2016 - 12:23 in Số học

Câu 1 Cho p là số nguyên tố lẻ, cm : $\sum_{k=1}^{p} C _{p}^{k} C_{p+k}^{k}-(2^{p}+1)\vdots p^{2}$
Câu 2: Cmr $\forall$ số tự nhiên n, phân số sau đây tối giản : $\frac{21n+4}{14n+3}$
Câu 3:$B=\frac{1}{630}x^{9} -\frac{1}{27}x^{7}+\frac{13}{30}x^{5}-\frac{82}{63}x^{3}+\frac{32}{35}x$
Cmr $B\in Z \forall x\in Z$

Câu 4: Cmr với mọi số nguyên a;b
(3a+5b;8a+13b)=(a;b)




#661848 $\frac{21n+4}{14n+3}$

Posted by sanghamhoc on 13-11-2016 - 23:53 in Số học

Câu 1 Cho p là số nguyên tố lẻ, cm : $\sum_{k=1}^{p} C _{p}^{k} C_{p+k}^{k}-(2^{p}+1)\vdots p^{2}$
Câu 2: Cmr $\forall$ số tự nhiên n, phân số sau đây tối giản : $\frac{21n+4}{14n+3}$
Câu 3:$B=\frac{1}{630}x^{9} -\frac{1}{27}x^{7}+\frac{13}{30}x^{5}-\frac{82}{63}x^{3}+\frac{32}{35}x$
Cmr $B\in Z \forall x\in Z$

Câu 4: Cmr với mọi số nguyên a;b
(3a+5b;8a+13b)=(a;b)




#660408 $2^{2^{10n+1}}+19 \vdots 23$

Posted by sanghamhoc on 03-11-2016 - 10:16 in Số học

Bài 1:Cmr $\forall n\in N$* ta có: $2^{2^{10n+1}}+19 \vdots 23$
Bài 2: Tìm số dư trong phép chia $51200^{2^{100}}$ chia cho 41

Bài 3: Cmr $0,3(1983^{1983}-1917^{1917})$ là số nguyên

Bài 4: Cmr $\sum_{k=1}^{26}k.10^{3k}\vdots 13$ với $k\in N$

Bài 5: Tìm $n\in N$ để $n^{n+1}(n+1)^{n}\vdots 5$

Bài 6: Giả sử p là số nguyên tố có dạng 3n+2 ($n\in N$)

cmr không tồn tại số nguyên x sao cho $x^{2}+3\vdots p$




#657550 $3\overrightarrow{MA} + 4\overrightarrow{MB...

Posted by sanghamhoc on 11-10-2016 - 21:12 in Hình học phẳng

Cho tam giác ABC có trọng tâm G, lấy các điểm M,N thỏa : 
$3\overrightarrow{MA} + 4\overrightarrow{MB}=\vec{0}$
$\overrightarrow{BC}=2\overrightarrow{CN}$
CMR: 3 điểm G,M,N thẳng hàng 




#655659 Tìm công thức tổng quát tính số tập hợp con của 1 tập hợp. ( theo cách quy nạp )

Posted by sanghamhoc on 26-09-2016 - 20:44 in Mệnh đề - tập hợp

Tìm công thức tổng quát tính số tập hợp con của 1 tập hợp. ( theo cách quy nạp ) 




#653795 $(x+3)\sqrt{-x^{2}-8x+48}=x-27$

Posted by sanghamhoc on 11-09-2016 - 20:37 in Phương trình - hệ phương trình - bất phương trình

Vui lòng để lại phản hồi cho từng bài mà mình đã đưa ra Hint nhen!

 

PT thứ nhất có thể xem như PT bậc hai theo ẩn $t=\sqrt{x^2+2}$. Hãy thử tính $\Delta.$

 

Nhận xét về phần trước căn và trong căn có gì đặc biệt..., nghĩa là tìm mối liên hệ giữa $A, B $ trong $A\sqrt{B}$. Từ đó giúp ta có cách giải.

 

 

 

 

Bài 2:

PT thứ nhất là PT thuần nhất... thử "so sánh" VT và VP!

anh ơi ở bài 2 : pt đầu tiên e giải được x=y rồi tiếp pt thứ 2 thay vào làm sao ạ ?